Expectation property of moment generating function











up vote
0
down vote

favorite












Let $X$ be a non-negative random variable and define $M_X(t) := mathbb{E}[e^{tX}]$. Suppose that $M_X(t) < infty mbox{ for all } t < alpha$, where $alpha > 0$.



It can be shown that this condition implies $mathbb{E}[X^k]<infty$ for all $k$, i.e. all moments exist.



How can we establish that $mathbb{E}[X^ke^{tX}] < infty$ for all $k$ and $ t < alpha$?










share|cite|improve this question






















  • My idea is to use tail property. Any expectation is integral (supposedly either in Stieltjes or in Lebesgue sense.) So you can divide your last expectation into the parts $[0 , epsilon]$ and $[epsilon , infty)$. And then analyse the convergence properties. (In the first region it is for sure bounded), In tail regions it can be done intuitively. $e^{x}$ is faster then $dF_{X}(x)$ in tail, so $dF_{X}(x)= o(e^{-x})$, such that for no $k$, $e^{x}*x^{k}$ can be faster $dF_{X}(x)$.(This is acc. proof, when you have continuous random variable, but I'm not sure with general case(can have jumps)).
    – kolobokish
    Nov 22 at 20:51

















up vote
0
down vote

favorite












Let $X$ be a non-negative random variable and define $M_X(t) := mathbb{E}[e^{tX}]$. Suppose that $M_X(t) < infty mbox{ for all } t < alpha$, where $alpha > 0$.



It can be shown that this condition implies $mathbb{E}[X^k]<infty$ for all $k$, i.e. all moments exist.



How can we establish that $mathbb{E}[X^ke^{tX}] < infty$ for all $k$ and $ t < alpha$?










share|cite|improve this question






















  • My idea is to use tail property. Any expectation is integral (supposedly either in Stieltjes or in Lebesgue sense.) So you can divide your last expectation into the parts $[0 , epsilon]$ and $[epsilon , infty)$. And then analyse the convergence properties. (In the first region it is for sure bounded), In tail regions it can be done intuitively. $e^{x}$ is faster then $dF_{X}(x)$ in tail, so $dF_{X}(x)= o(e^{-x})$, such that for no $k$, $e^{x}*x^{k}$ can be faster $dF_{X}(x)$.(This is acc. proof, when you have continuous random variable, but I'm not sure with general case(can have jumps)).
    – kolobokish
    Nov 22 at 20:51















up vote
0
down vote

favorite









up vote
0
down vote

favorite











Let $X$ be a non-negative random variable and define $M_X(t) := mathbb{E}[e^{tX}]$. Suppose that $M_X(t) < infty mbox{ for all } t < alpha$, where $alpha > 0$.



It can be shown that this condition implies $mathbb{E}[X^k]<infty$ for all $k$, i.e. all moments exist.



How can we establish that $mathbb{E}[X^ke^{tX}] < infty$ for all $k$ and $ t < alpha$?










share|cite|improve this question













Let $X$ be a non-negative random variable and define $M_X(t) := mathbb{E}[e^{tX}]$. Suppose that $M_X(t) < infty mbox{ for all } t < alpha$, where $alpha > 0$.



It can be shown that this condition implies $mathbb{E}[X^k]<infty$ for all $k$, i.e. all moments exist.



How can we establish that $mathbb{E}[X^ke^{tX}] < infty$ for all $k$ and $ t < alpha$?







probability probability-theory random-variables moment-generating-functions expected-value






share|cite|improve this question













share|cite|improve this question











share|cite|improve this question




share|cite|improve this question










asked Nov 22 at 20:36









jesterII

1,19121226




1,19121226












  • My idea is to use tail property. Any expectation is integral (supposedly either in Stieltjes or in Lebesgue sense.) So you can divide your last expectation into the parts $[0 , epsilon]$ and $[epsilon , infty)$. And then analyse the convergence properties. (In the first region it is for sure bounded), In tail regions it can be done intuitively. $e^{x}$ is faster then $dF_{X}(x)$ in tail, so $dF_{X}(x)= o(e^{-x})$, such that for no $k$, $e^{x}*x^{k}$ can be faster $dF_{X}(x)$.(This is acc. proof, when you have continuous random variable, but I'm not sure with general case(can have jumps)).
    – kolobokish
    Nov 22 at 20:51




















  • My idea is to use tail property. Any expectation is integral (supposedly either in Stieltjes or in Lebesgue sense.) So you can divide your last expectation into the parts $[0 , epsilon]$ and $[epsilon , infty)$. And then analyse the convergence properties. (In the first region it is for sure bounded), In tail regions it can be done intuitively. $e^{x}$ is faster then $dF_{X}(x)$ in tail, so $dF_{X}(x)= o(e^{-x})$, such that for no $k$, $e^{x}*x^{k}$ can be faster $dF_{X}(x)$.(This is acc. proof, when you have continuous random variable, but I'm not sure with general case(can have jumps)).
    – kolobokish
    Nov 22 at 20:51


















My idea is to use tail property. Any expectation is integral (supposedly either in Stieltjes or in Lebesgue sense.) So you can divide your last expectation into the parts $[0 , epsilon]$ and $[epsilon , infty)$. And then analyse the convergence properties. (In the first region it is for sure bounded), In tail regions it can be done intuitively. $e^{x}$ is faster then $dF_{X}(x)$ in tail, so $dF_{X}(x)= o(e^{-x})$, such that for no $k$, $e^{x}*x^{k}$ can be faster $dF_{X}(x)$.(This is acc. proof, when you have continuous random variable, but I'm not sure with general case(can have jumps)).
– kolobokish
Nov 22 at 20:51






My idea is to use tail property. Any expectation is integral (supposedly either in Stieltjes or in Lebesgue sense.) So you can divide your last expectation into the parts $[0 , epsilon]$ and $[epsilon , infty)$. And then analyse the convergence properties. (In the first region it is for sure bounded), In tail regions it can be done intuitively. $e^{x}$ is faster then $dF_{X}(x)$ in tail, so $dF_{X}(x)= o(e^{-x})$, such that for no $k$, $e^{x}*x^{k}$ can be faster $dF_{X}(x)$.(This is acc. proof, when you have continuous random variable, but I'm not sure with general case(can have jumps)).
– kolobokish
Nov 22 at 20:51












1 Answer
1






active

oldest

votes

















up vote
2
down vote



accepted










Let $t<t'<alpha$. Then $e^{(t'-t)x} geq frac {(t'-t)^{k}x^{k}} {k!}$ so $x^{k}e^{tx} leq frac {e^{t'x} k!} {(t'-t)^{k}}$. Hence $EX^{k}e^{tX}leq M(t') frac { k!} {(t'-t)^{k}}<infty$.






share|cite|improve this answer





















    Your Answer





    StackExchange.ifUsing("editor", function () {
    return StackExchange.using("mathjaxEditing", function () {
    StackExchange.MarkdownEditor.creationCallbacks.add(function (editor, postfix) {
    StackExchange.mathjaxEditing.prepareWmdForMathJax(editor, postfix, [["$", "$"], ["\\(","\\)"]]);
    });
    });
    }, "mathjax-editing");

    StackExchange.ready(function() {
    var channelOptions = {
    tags: "".split(" "),
    id: "69"
    };
    initTagRenderer("".split(" "), "".split(" "), channelOptions);

    StackExchange.using("externalEditor", function() {
    // Have to fire editor after snippets, if snippets enabled
    if (StackExchange.settings.snippets.snippetsEnabled) {
    StackExchange.using("snippets", function() {
    createEditor();
    });
    }
    else {
    createEditor();
    }
    });

    function createEditor() {
    StackExchange.prepareEditor({
    heartbeatType: 'answer',
    convertImagesToLinks: true,
    noModals: true,
    showLowRepImageUploadWarning: true,
    reputationToPostImages: 10,
    bindNavPrevention: true,
    postfix: "",
    imageUploader: {
    brandingHtml: "Powered by u003ca class="icon-imgur-white" href="https://imgur.com/"u003eu003c/au003e",
    contentPolicyHtml: "User contributions licensed under u003ca href="https://creativecommons.org/licenses/by-sa/3.0/"u003ecc by-sa 3.0 with attribution requiredu003c/au003e u003ca href="https://stackoverflow.com/legal/content-policy"u003e(content policy)u003c/au003e",
    allowUrls: true
    },
    noCode: true, onDemand: true,
    discardSelector: ".discard-answer"
    ,immediatelyShowMarkdownHelp:true
    });


    }
    });














    draft saved

    draft discarded


















    StackExchange.ready(
    function () {
    StackExchange.openid.initPostLogin('.new-post-login', 'https%3a%2f%2fmath.stackexchange.com%2fquestions%2f3009628%2fexpectation-property-of-moment-generating-function%23new-answer', 'question_page');
    }
    );

    Post as a guest















    Required, but never shown

























    1 Answer
    1






    active

    oldest

    votes








    1 Answer
    1






    active

    oldest

    votes









    active

    oldest

    votes






    active

    oldest

    votes








    up vote
    2
    down vote



    accepted










    Let $t<t'<alpha$. Then $e^{(t'-t)x} geq frac {(t'-t)^{k}x^{k}} {k!}$ so $x^{k}e^{tx} leq frac {e^{t'x} k!} {(t'-t)^{k}}$. Hence $EX^{k}e^{tX}leq M(t') frac { k!} {(t'-t)^{k}}<infty$.






    share|cite|improve this answer

























      up vote
      2
      down vote



      accepted










      Let $t<t'<alpha$. Then $e^{(t'-t)x} geq frac {(t'-t)^{k}x^{k}} {k!}$ so $x^{k}e^{tx} leq frac {e^{t'x} k!} {(t'-t)^{k}}$. Hence $EX^{k}e^{tX}leq M(t') frac { k!} {(t'-t)^{k}}<infty$.






      share|cite|improve this answer























        up vote
        2
        down vote



        accepted







        up vote
        2
        down vote



        accepted






        Let $t<t'<alpha$. Then $e^{(t'-t)x} geq frac {(t'-t)^{k}x^{k}} {k!}$ so $x^{k}e^{tx} leq frac {e^{t'x} k!} {(t'-t)^{k}}$. Hence $EX^{k}e^{tX}leq M(t') frac { k!} {(t'-t)^{k}}<infty$.






        share|cite|improve this answer












        Let $t<t'<alpha$. Then $e^{(t'-t)x} geq frac {(t'-t)^{k}x^{k}} {k!}$ so $x^{k}e^{tx} leq frac {e^{t'x} k!} {(t'-t)^{k}}$. Hence $EX^{k}e^{tX}leq M(t') frac { k!} {(t'-t)^{k}}<infty$.







        share|cite|improve this answer












        share|cite|improve this answer



        share|cite|improve this answer










        answered Nov 22 at 23:50









        Kavi Rama Murthy

        47.3k31854




        47.3k31854






























            draft saved

            draft discarded




















































            Thanks for contributing an answer to Mathematics Stack Exchange!


            • Please be sure to answer the question. Provide details and share your research!

            But avoid



            • Asking for help, clarification, or responding to other answers.

            • Making statements based on opinion; back them up with references or personal experience.


            Use MathJax to format equations. MathJax reference.


            To learn more, see our tips on writing great answers.





            Some of your past answers have not been well-received, and you're in danger of being blocked from answering.


            Please pay close attention to the following guidance:


            • Please be sure to answer the question. Provide details and share your research!

            But avoid



            • Asking for help, clarification, or responding to other answers.

            • Making statements based on opinion; back them up with references or personal experience.


            To learn more, see our tips on writing great answers.




            draft saved


            draft discarded














            StackExchange.ready(
            function () {
            StackExchange.openid.initPostLogin('.new-post-login', 'https%3a%2f%2fmath.stackexchange.com%2fquestions%2f3009628%2fexpectation-property-of-moment-generating-function%23new-answer', 'question_page');
            }
            );

            Post as a guest















            Required, but never shown





















































            Required, but never shown














            Required, but never shown












            Required, but never shown







            Required, but never shown

































            Required, but never shown














            Required, but never shown












            Required, but never shown







            Required, but never shown







            Popular posts from this blog

            Quarter-circle Tiles

            build a pushdown automaton that recognizes the reverse language of a given pushdown automaton?

            Mont Emei